K
Khách

Hãy nhập câu hỏi của bạn vào đây, nếu là tài khoản VIP, bạn sẽ được ưu tiên trả lời.

22 tháng 4 2018

số đó là 9 chúc học tốt

3 tháng 8 2015

Theo mình thì phân tích ra thành thế này
gọi số cần tìm là \(ab\) có:
\(ab=x^3;a+b=x^2\)(\(x\) là số tự nhiên mà khi lập phương lên thì bằng \(ab\), khi bình phương lên thì bằng \(a+b\))
Từ đó ta có: \(10a+b=x^3\)
\(a+b=x^2\)
Rồi suy ra được ab thì phải, mình không biết có đúng không nữa, nếu mà các bước mình làm đúng thì bạn nghiên cứu thêm nhé

3 tháng 8 2015

Bạn ơi, cái này mình cũng làm ra đến đó rồi nhưng mà chưa biết làm tiếp. Bạn giúp mình nhé

13 tháng 5 2017

a) 0 ; 1

b) 0 ; 1 ; -1

18 tháng 2 2016

a. ta thấy các số từ 2 bình phương trở lên là số lớn hơn chính nó len số cần tìm sẽ nhỏ hơn 2 vậy ta kết luận số cần tim là 0 và 1

b. tương tự như phần a.

18 tháng 2 2016

a. số 1 và 0

b) số 1, -1, 0

1 tháng 4 2018

Số đó là 0 và 1  ko tin thì thử xem

1 tháng 4 2018

Gọi số đó là (ab) 
(ab)^2=(a+b)^3 
Từ đó suy ra (ab) phải là lập phương của 1 số, a+b là bình phương của 1 số 
(ab) = 27 hoặc 64 
chỉ có 27 thỏa mãn 
vậy (ab)=27

1 tháng 9 2020

Gọi số điểm của tổ 1 là a ; số điểm của tổ 2 là b ; số điểm của tổ 3 là c (a;b;c .> 0)

Ta có \(\frac{a}{3}=\frac{b}{4}=\frac{c}{2}\)

Đặt \(\frac{a}{3}=\frac{b}{4}=\frac{c}{2}=k\Rightarrow\hept{\begin{cases}a=3k\\b=4k\\c=2k\end{cases}}\)

Lại có 5a2 + 7c2 - b2= 1282500

<=> 5(3k)2 - (4k)2 + 7(2k)2 = 1282500

=> 45k2 - 16k2 + 28k2 = 1282500

=> k2(45 - 16 + 28) = 1282500

=> k2.57 = 1282500

=> k2 = 22500

=> k2 = 1502

=> k = \(\pm\)150

=> k = 150 (Vì a ; b ; c > 0)

Khi k = 150 => a = 450 ; b = 600; c = 300

Vậy nhóm 1 có 450 điểm ; nhóm 2 có 600 điểm ; nhóm 3 co 300 điểm

2) Đặt \(\frac{a}{b}=\frac{c}{d}=k\Rightarrow\hept{\begin{cases}a=bk\\c=dk\end{cases}}\)

Khi đó \(\frac{11a+7b}{11a-7b}=\frac{11bk+7b}{11bk-7b}=\frac{b\left(11k+7\right)}{b\left(11k-7\right)}=\frac{11k+7}{11k-7}\left(1\right)\);

\(\frac{11c+7d}{11c-7d}=\frac{11dk+7d}{11dk-7d}=\frac{d\left(11k+7\right)}{d\left(11k-7\right)}=\frac{11k+7}{11k-7}\left(2\right)\)

Từ (1) (2) => \(\frac{11a+7b}{11a-7b}=\frac{11c+7d}{11c-7d}\)(đpcm)